[Physics] Find Eigenstates of a Hamiltonian that lets two spin 1/2 interact but also acts on one of them

hamiltonian-formalismquantum mechanicsquantum-spin

I have the following Hamiltonian describing two spin 1/2 systems, represented by the pauli matrices $\sigma_1$ and $\sigma_2$:

$H = D \sigma_{1z} + J (\sigma_1 \cdot \sigma_2) $.

The two spins are coupled by $J$ but at the same time the first spin $\sigma_1$ is also under
the influence of $D$ that influences it's z-component. My questions now are:


  1. Is this Hamiltonian even allowed in a physical way? I.e. can I just "divide" the spin
    $\sigma_1$ like this and claim that on the one hand it is interacting with another spin but on the other hand it is also influenced by something in which I do not take the second spin into account anymore and just add the Hamiltonians up. Somehow I feel that a perturbation is more appropiate.

  2. Now of course I want to find the eigenstates and eigenvalues of this Hamiltonian, for this I couple the spins and try the following 4×4 Hamiltonian with the new 4×4 spin operators:
    $$H = D (\sigma_{1z} \otimes \mathbb{1}^{(2×2)}) + J (\sigma_{1x} \otimes \sigma_{2x} + \sigma_{1y} \otimes \sigma_{2y} + \sigma_{1z} \otimes \sigma_{2z}) $$
    I then naively diagonalize this Hamiltonian with Matlab and get some eigenvalues. The eigenvalues seem to agree with the expected energies. But my problem now is that I'm not sure if I this is correct based on my first question and I have no idea what the eigenstates are supposed to mean, I feel like I need to do a unitary transformation or a base change to make sense out of the eigenstates that I get. I also can't find operators that now commute with this Hamiltonian, i.e. $\sigma^2$ does not commute anymore.
    Where I have used new operator for the total spin in the following form (here only for the z-component):
    $$S_z = \sigma_{1z} \otimes \mathbb{1}^{(2×2)} + \mathbb{1}^{(2×2)} \otimes \sigma_{2z}. $$
    Is there a way to find good quantum numbers again that are conserved since I'm really confused now how I am supposed to label the eigenstates, i.e. which m_z value and total spin they have.

Thanks!

Best Answer

I'm not so sure, if this is really, what you're looking for, but you can of course solve this easy problem analytically.

To do this, it is clever to first analyze the easier Hamiltonian $H_0 = 2g (\vec L \cdot \vec S)$, where the $L_i$ and $S_j$ fulfill independent $SU(2)$-algebrae $$ [L_i, L_j] = i \epsilon_{ijk} L_k\\ [S_i, S_j] = i \epsilon_{ijk} S_k. $$ This Hamiltonian can be written as $$H_0 = g(J^2 - L^2 - S^2),$$ where we have defined the following operators: $$L^2 = \sum_{i=1}^3 L_i^2 \otimes \mathbb{1},\\ S^2 = \sum_{i=1}^3 \mathbb{1} \otimes S_i^2, \\ J_i = L_i\otimes \mathbb{1} + \mathbb{1}\otimes S_i,\\J^2 = \sum_{i=1}^3 J_i^2.$$ Now as $L^2$ and $S^2$ are equal to $\frac{1}{2} (1+\frac{1}{2})$ on the subspace we are interested in (namely the one of a spin-1/2-particle), we can write the Hamiltonian as $$H_0 = g(J^2 - 3/2)$$ and by simple addition of angular momenta, we find the eigenstates $|j, m\rangle$: $$|1, 1\rangle = \left|\uparrow\uparrow\right\rangle\\ |1, 0\rangle = \frac{1}{\sqrt{2}}(\left|\uparrow\downarrow\right\rangle+\left|\downarrow\uparrow\right\rangle) \\ |1, -1\rangle = \left|\downarrow\downarrow\right\rangle\\ |0, 0\rangle = \frac{1}{\sqrt{2}}(\left|\uparrow\downarrow\right\rangle-\left|\downarrow\uparrow\right\rangle)$$ with energies $E_0 = -3g/2$ and $E_1 = g/2$ respectively (and obvious notation for the product base of the two particle hilbert space).

Now let's proceed to the real problem and add the second term. As you said, $$[J^2, L_z] = 2i (\vec L\times \vec S)_z$$ and so the eigenvalue of $J^2$ is no good quantum number anymore. But $$[H, J_z] = 0,$$ so we can label the states of the system by their energy and their $J_z$ component. Indeed, if we calculate the action of $H$ on the former eigenbase, we see, that it only mixes $|1, 0\rangle$ and $|0, 0\rangle$. By diagonalizing the full Hamiltonian we find a new base of eigenstates:

$$|m=\pm 1, E_{\pm1} \rangle = |1, \pm 1\rangle\\ |m=0, E_{0,\pm}\rangle = \frac{1}{C_\pm}\left(\left(g\pm\sqrt{g^2+d^2}\right) |1, 0\rangle + d|0, 0\rangle\right),$$ where $C_\pm^2 = 2\left(g^2 \pm g \sqrt{g^2+d^2} + d^2\right)$ and corresponding energies $$E_{\pm1} = \frac{g}{2}\pm d\\ E_{0, \pm} = -\frac{g}{2} \pm \sqrt{g^2 + d^2},$$ which of course reduce to the eigenvalues of $H_0$ if you turn $d$ to zero.